Essential University Physics: Volume 1 (3rd Edition)

Published by Pearson
ISBN 10: 0321993721
ISBN 13: 978-0-32199-372-4

Chapter 19 - Exercises and Problems - Page 351: 62

Answer

a) $T_h = \frac{T_{h0}}{e^({\frac{P_0t}{-mcT_c+mcT_{h0}}})}$ b)$t=\frac{(-mcT_c+mcT_{h0})ln(\frac{T_{h0}}{T_c})}{P_0}$

Work Step by Step

a) We know the following equations: $P=P_0\frac{T_h-T_c}{T_{h0}-T_c}$ $dW = Pdt$ $Pdt = \frac{(-mc\space dT_h)(T_h-T_c)}{T_h}$ Combining these two equations and taking the integral, we simplify to find that the value of $T_h$ is: $T_h = \frac{T_{h0}}{e^({\frac{P_0t}{-mcT_c+mcT_{h0}}})}$ b) First, we simplify the above expression for t: $T_h = \frac{T_{h0}}{e^({\frac{P_0t}{-mcT_c+mcT_{h0}}})}$ $T_h(e^{\frac{P_0t}{-mcT_c+mcT_{h0}}}) = T_{h0}$ $(e^{\frac{P_0t}{-mcT_c+mcT_{h0}}}) = \frac{T_{h0}}{T_h}$ $\frac{P_0t}{-mcT_c+mcT_{h0}}=ln(\frac{T_{h0}}{T_h})$ $P_0t=(-mcT_c+mcT_{h0})ln(\frac{T_{h0}}{T_h})$ $t=\frac{(-mcT_c+mcT_{h0})ln(\frac{T_{h0}}{T_h})}{P_0}$ We know that the engine will not longer work when $T_h$ and $T_c$ are equal, so it follows: $t=\frac{(-mcT_c+mcT_{h0})ln(\frac{T_{h0}}{T_c})}{P_0}$
Update this answer!

You can help us out by revising, improving and updating this answer.

Update this answer

After you claim an answer you’ll have 24 hours to send in a draft. An editor will review the submission and either publish your submission or provide feedback.